Difference between revisions of "2014 AMC 12A Problems/Problem 1"

(Solution)
(Replaced content with "Deleted because of duplicate.")
Line 1: Line 1:
==Problem ==
+
Deleted because of duplicate.
 
 
What is <math>10 \cdot \left(\tfrac{1}{2} + \tfrac{1}{5} + \tfrac{1}{10}\right)^{-1}?</math>
 
 
 
<math> \textbf{(A)}\ 3\qquad\textbf{(B)}\ 8\qquad\textbf{(C)}\ \frac{25}{2}\qquad\textbf{(D)}}\ \frac{170}{3}\qquad\textbf{(E)}\ 170</math>
 
 
 
== Solution ==
 
Sum the fractions over the common denominator: <math>\dfrac{1}{2}+\dfrac15+\dfrac1{10}=\dfrac{5+2+1}{10}=\dfrac45</math>
 
 
 
Now the answer is just some arithmetic: <math>10*\left(\dfrac45\right)^{-1}=10*\dfrac{5}{4}= \boxed{\textbf{(C)}\ \dfrac{25}2}</math>
 
 
 
==See Also==
 
 
 
{{AMC12 box|year=2014|ab=A|before=First Problem|num-a=2}}
 
{{MAA Notice}}
 

Revision as of 20:16, 7 February 2014

Deleted because of duplicate.